¿Qué le hace al hamiltoniano HHH una simetría que cambia el lagrangiano por una derivada total?

Una pequeña transformación de simetría puede cambiar el Lagrangiano L por una derivada temporal total de alguna función F . Este es un hecho básico utilizado en la demostración del teorema de Noether.

¿Cómo podemos ver el efecto de este término derivado total en el marco hamiltoniano? ¿Hay un buen ejemplo para trabajar? No puedo pensar en uno fuera de mi cabeza. Simplemente me parece extraño que todo este alboroto sobre las derivadas totales parezca desaparecer en el marco hamiltoniano.

Respuestas (3)

i) Descargo de responsabilidad. Como purista, desaprobé la praxis común de llamar a la implicación

(1) { q , H } + q t   =   0 d q d t     0.
para una 'versión hamiltoniana del teorema de Noether', cf. mi Phys.SE responde aquí y aquí . Mi razón es que la implicación (1) es simplemente una consecuencia trivial de las ecuaciones de Hamilton, nada más.

II) En cambio, una 'versión hamiltoniana del teorema de Noether' debería referirse a cuasi-simetrías de una acción hamiltoniana

(2) S H [ q , pag ]   :=   d t   L H ( q , q ˙ , pag , t ) ,
y sus correspondientes leyes de conservación. Aquí L H es el llamado hamiltoniano lagrangiano
(3) L H ( q , q ˙ , pag , t )   :=   i = 1 norte pag i q ˙ i H ( q , pag , t ) .

III) Es un malentendido que todo ese alboroto sobre las derivadas totales [...] desaparezca en el marco hamiltoniano. La versión hamiltoniana permite que la acción hamiltoniana solo sea invariante hasta los términos de contorno (es decir, la llamada cuasisimetría ) al igual que en la formulación lagrangiana estándar del teorema de Noether . Consulte también esta publicación Phys.SE relacionada.

Supongo que mi pregunta podría entonces refinarse: en el marco hamiltoniano, ¿existe una diferencia fundamental entre una simetría para la cual d L = 0 y uno por el cual d L = F ˙ ?
Solo necesitas la última condición para el teorema de Noether.

Supongo que descubrí la "respuesta" a mi pregunta muy vaga, aunque las otras respuestas aquí también son útiles. El "Hamiltoniano Lagrangiano" es

L = pag i q ˙ i H .
Digamos que tenemos una carga conservada q , eso es
{ q , H } = 0.
Si hacemos la pequeña variación de simetría
d q i = q pag i d pag i = q q i
entonces
d L = q q i q ˙ i pag i d d t ( q pag i ) + { H , q } = q pag i q ˙ i pag ˙ i q pag i + d d t ( pag i q pag i ) = d d t ( pag i q pag i q )

Entonces podemos ver que L cambia necesariamente por una derivada total. cuando la cantidad pag i q pag i q = 0 , la derivada total es 0 . Esto sucede cuando la cantidad conservada es de la forma

q = pag i F i ( q ) .
Nótese que en el caso anterior,
d q i = F i ( q )
Es decir, transformaciones de simetría que no "confunden" el pag está con el q 's no tienen ningún término derivado total en d L .

La razón por la que no hablamos de "cambiar el hamiltoniano por una derivada total" es que las simetrías y las leyes de conservación generalmente se manejan de manera diferente en el cuadro hamiltoniano.

En mecánica hamiltoniana, cualquier función F en el espacio de fase genera un flujo en el espacio de fase, es decir, una familia de transformaciones canónicas de un parámetro ( q , pag ) ( q ~ ( α ) , pag ~ ( α ) ) . La tasa de cambio inducida de cualquier otra función de espacio de fase gramo es

d gramo d α = { gramo , F } .
En particular, el propio hamiltoniano genera la traducción del tiempo,
d gramo d t = { gramo , H } .
La declaración de que q ( q , pag ) es una cantidad conservada es simplemente
{ q , H } = 0.
Es decir, la evolución temporal generada por H no cambia el valor de q . La clave es que esto es equivalente, por la antisimetría del corchete de Poisson, a { H , q } = 0 , que establece que las transformaciones canónicas generadas por q no cambies los valores de H .

Así, dada una transformación canónica infinitesimal que mantiene H lo mismo, su generador es una cantidad conservada. Esto es lo más parecido al teorema de Noether que normalmente verás en la mecánica hamiltoniana. ya que se refiere solo H , no la integral de H , no hay necesidad de hablar de mantener H invariante hasta una derivada total, solo tiene que ser invariante, punto. (Pero también vea la respuesta de Qmechanic, sobre una formulación similar a una acción donde sí aparece).